Multiplying summation with same indices and limits











up vote
1
down vote

favorite












What would be
$(1-sum limits_{k=0}^m x^k )(1-sum limits_{k=0}^m y^k ) ?$



I dont understand how can I multiply summation of same indices. I checked "multiplication of finite sum (inner product space)" this post but it is different than my case.



Any suggestion?










share|cite|improve this question






















  • These are both just partial sums of a geometric series, and $$sum_{k = 0}^m x^k = frac{1 - x^{m + 1}}{1 - x}.$$
    – T. Bongers
    yesterday










  • k is simply a dummy. For either sum any letter could have been used. The k's in the two sums have no relation to each other.
    – herb steinberg
    yesterday










  • As per my case, both summation have same k and same limits. I am not sure how to evaluate $sum limits_{k=0}^{m}sum limits_{k=0}^{m}x^ky^k$
    – hakkunamattata
    yesterday










  • @hakkunamattata As herb states above, it is not the "same" $k$. The summation indices are dummy variables, $sum_{k=0}^m x^k = sum_{j=0}^m x^k=sum_{heartsuit=0}^m x^{heartsuit}.$So $$left(sum_{k=0}^m x^kright)left(sum_{k=0}^m y^kright)=left(sum_{k=0}^m x^kright)left(sum_{ell=0}^m y^ellright)=sum_{k=0}^m sum_{ell=0}^m x^ky^ell$$
    – Clement C.
    yesterday












  • I suggest you work this out by hand for $m=2$ or $3$. Often summations with $Sigma$ are clearer if you do a small special case. For infinite sums write $a_1 + a_2 + cdots$.
    – Ethan Bolker
    yesterday















up vote
1
down vote

favorite












What would be
$(1-sum limits_{k=0}^m x^k )(1-sum limits_{k=0}^m y^k ) ?$



I dont understand how can I multiply summation of same indices. I checked "multiplication of finite sum (inner product space)" this post but it is different than my case.



Any suggestion?










share|cite|improve this question






















  • These are both just partial sums of a geometric series, and $$sum_{k = 0}^m x^k = frac{1 - x^{m + 1}}{1 - x}.$$
    – T. Bongers
    yesterday










  • k is simply a dummy. For either sum any letter could have been used. The k's in the two sums have no relation to each other.
    – herb steinberg
    yesterday










  • As per my case, both summation have same k and same limits. I am not sure how to evaluate $sum limits_{k=0}^{m}sum limits_{k=0}^{m}x^ky^k$
    – hakkunamattata
    yesterday










  • @hakkunamattata As herb states above, it is not the "same" $k$. The summation indices are dummy variables, $sum_{k=0}^m x^k = sum_{j=0}^m x^k=sum_{heartsuit=0}^m x^{heartsuit}.$So $$left(sum_{k=0}^m x^kright)left(sum_{k=0}^m y^kright)=left(sum_{k=0}^m x^kright)left(sum_{ell=0}^m y^ellright)=sum_{k=0}^m sum_{ell=0}^m x^ky^ell$$
    – Clement C.
    yesterday












  • I suggest you work this out by hand for $m=2$ or $3$. Often summations with $Sigma$ are clearer if you do a small special case. For infinite sums write $a_1 + a_2 + cdots$.
    – Ethan Bolker
    yesterday













up vote
1
down vote

favorite









up vote
1
down vote

favorite











What would be
$(1-sum limits_{k=0}^m x^k )(1-sum limits_{k=0}^m y^k ) ?$



I dont understand how can I multiply summation of same indices. I checked "multiplication of finite sum (inner product space)" this post but it is different than my case.



Any suggestion?










share|cite|improve this question













What would be
$(1-sum limits_{k=0}^m x^k )(1-sum limits_{k=0}^m y^k ) ?$



I dont understand how can I multiply summation of same indices. I checked "multiplication of finite sum (inner product space)" this post but it is different than my case.



Any suggestion?







summation summation-method






share|cite|improve this question













share|cite|improve this question











share|cite|improve this question




share|cite|improve this question










asked yesterday









hakkunamattata

454




454












  • These are both just partial sums of a geometric series, and $$sum_{k = 0}^m x^k = frac{1 - x^{m + 1}}{1 - x}.$$
    – T. Bongers
    yesterday










  • k is simply a dummy. For either sum any letter could have been used. The k's in the two sums have no relation to each other.
    – herb steinberg
    yesterday










  • As per my case, both summation have same k and same limits. I am not sure how to evaluate $sum limits_{k=0}^{m}sum limits_{k=0}^{m}x^ky^k$
    – hakkunamattata
    yesterday










  • @hakkunamattata As herb states above, it is not the "same" $k$. The summation indices are dummy variables, $sum_{k=0}^m x^k = sum_{j=0}^m x^k=sum_{heartsuit=0}^m x^{heartsuit}.$So $$left(sum_{k=0}^m x^kright)left(sum_{k=0}^m y^kright)=left(sum_{k=0}^m x^kright)left(sum_{ell=0}^m y^ellright)=sum_{k=0}^m sum_{ell=0}^m x^ky^ell$$
    – Clement C.
    yesterday












  • I suggest you work this out by hand for $m=2$ or $3$. Often summations with $Sigma$ are clearer if you do a small special case. For infinite sums write $a_1 + a_2 + cdots$.
    – Ethan Bolker
    yesterday


















  • These are both just partial sums of a geometric series, and $$sum_{k = 0}^m x^k = frac{1 - x^{m + 1}}{1 - x}.$$
    – T. Bongers
    yesterday










  • k is simply a dummy. For either sum any letter could have been used. The k's in the two sums have no relation to each other.
    – herb steinberg
    yesterday










  • As per my case, both summation have same k and same limits. I am not sure how to evaluate $sum limits_{k=0}^{m}sum limits_{k=0}^{m}x^ky^k$
    – hakkunamattata
    yesterday










  • @hakkunamattata As herb states above, it is not the "same" $k$. The summation indices are dummy variables, $sum_{k=0}^m x^k = sum_{j=0}^m x^k=sum_{heartsuit=0}^m x^{heartsuit}.$So $$left(sum_{k=0}^m x^kright)left(sum_{k=0}^m y^kright)=left(sum_{k=0}^m x^kright)left(sum_{ell=0}^m y^ellright)=sum_{k=0}^m sum_{ell=0}^m x^ky^ell$$
    – Clement C.
    yesterday












  • I suggest you work this out by hand for $m=2$ or $3$. Often summations with $Sigma$ are clearer if you do a small special case. For infinite sums write $a_1 + a_2 + cdots$.
    – Ethan Bolker
    yesterday
















These are both just partial sums of a geometric series, and $$sum_{k = 0}^m x^k = frac{1 - x^{m + 1}}{1 - x}.$$
– T. Bongers
yesterday




These are both just partial sums of a geometric series, and $$sum_{k = 0}^m x^k = frac{1 - x^{m + 1}}{1 - x}.$$
– T. Bongers
yesterday












k is simply a dummy. For either sum any letter could have been used. The k's in the two sums have no relation to each other.
– herb steinberg
yesterday




k is simply a dummy. For either sum any letter could have been used. The k's in the two sums have no relation to each other.
– herb steinberg
yesterday












As per my case, both summation have same k and same limits. I am not sure how to evaluate $sum limits_{k=0}^{m}sum limits_{k=0}^{m}x^ky^k$
– hakkunamattata
yesterday




As per my case, both summation have same k and same limits. I am not sure how to evaluate $sum limits_{k=0}^{m}sum limits_{k=0}^{m}x^ky^k$
– hakkunamattata
yesterday












@hakkunamattata As herb states above, it is not the "same" $k$. The summation indices are dummy variables, $sum_{k=0}^m x^k = sum_{j=0}^m x^k=sum_{heartsuit=0}^m x^{heartsuit}.$So $$left(sum_{k=0}^m x^kright)left(sum_{k=0}^m y^kright)=left(sum_{k=0}^m x^kright)left(sum_{ell=0}^m y^ellright)=sum_{k=0}^m sum_{ell=0}^m x^ky^ell$$
– Clement C.
yesterday






@hakkunamattata As herb states above, it is not the "same" $k$. The summation indices are dummy variables, $sum_{k=0}^m x^k = sum_{j=0}^m x^k=sum_{heartsuit=0}^m x^{heartsuit}.$So $$left(sum_{k=0}^m x^kright)left(sum_{k=0}^m y^kright)=left(sum_{k=0}^m x^kright)left(sum_{ell=0}^m y^ellright)=sum_{k=0}^m sum_{ell=0}^m x^ky^ell$$
– Clement C.
yesterday














I suggest you work this out by hand for $m=2$ or $3$. Often summations with $Sigma$ are clearer if you do a small special case. For infinite sums write $a_1 + a_2 + cdots$.
– Ethan Bolker
yesterday




I suggest you work this out by hand for $m=2$ or $3$. Often summations with $Sigma$ are clearer if you do a small special case. For infinite sums write $a_1 + a_2 + cdots$.
– Ethan Bolker
yesterday










2 Answers
2






active

oldest

votes

















up vote
0
down vote













Let's examine series multiplication first:
$$A=sum_{igeq0}a_i=a_0+a_1+a_2+dots$$
$$B=sum_{igeq0}b_i=b_0+b_1+b_2+dots$$
$$AB=big(a_0+a_1+dotsbig)B$$
$$AB=a_0S_2+big(a_1+a_2+dotsbig)B$$
$$AB=a_0S_2+a_1S_2+big(a_2+a_3+dotsbig)B$$
$$AB=a_0S_2+a_1S_2+a_2S_2+big(a_3+a_4+dotsbig)B$$
This pattern continues:
$$AB=a_0B+a_1B+a_2B+a_3B+dots$$
$$AB=sum_{igeq0}a_iB$$
Now note the following:
$$a_iB=a_isum_{kgeq0}b_k$$
$$a_iB=a_ibig(b_0+b_1+b_2+dotsbig)$$
$$a_iB=a_ib_0+a_ib_1+a_ib_2+dots$$
$$a_iB=sum_{kgeq0}a_ib_k$$
Plugging in:
$$AB=sum_{igeq0}sum_{kgeq0}a_ib_k$$
Since $i$ is independent of $k$, and they belong to the same set (namely ${xinBbb Z:xgeq0})$, we know that
$$
begin{align}
AB & = a_0b_0+a_0b_1+a_0b_2+dots \
& + a_1b_0+a_1b_1+a_1b_2+dots \
& + a_2b_0+a_2b_1+a_2b_2+dots \
& +dots
end{align}
$$

Which can be greatly abbreviated:
$$AB=sum_{i,kin S}a_ib_k$$
Where
$$S={xinBbb Z:xgeq0}={0,1,2,dots}$$





Now we can move onto something more related to your problem:
$$A=sum_{i=0}^{m}a_i$$
$$B=sum_{i=0}^{m}b_i$$
These are just like the case above:
$$AB=sum_{i,kin S}a_ib_k$$
Where $$S={xinBbb Z:0leq xleq m}$$
And of course the fact
$$(1-A)(1-B)=AB-A-B+1$$
still holds when $A$ and $B$ are series. But one should note:
$$-A=-a_0-a_1-a_2-dots$$
and not $$-A=-a_0+a_1+a_2+dots$$






share|cite|improve this answer




























    up vote
    0
    down vote













    The index variables $k$ are so-called bound variables. This means that their scope (i.e. range of validity) is determined by their sigma-operator $sum$ and the operator precedence rules.




    The following representations are valid
    begin{align*}
    left(1+sum_{k=0}^mx^kright)left(1+sum_{k=0}^my^kright)&=
    left(1+color{green}{left(sum_{k=0}^mx^kright)}right)left(1+color{blue}{left(sum_{k=0}^my^kright)}right)tag{1}\
    &=left(1+sum_{k=0}^mx^kright)left(1+sum_{color{blue}{j=0}}^my^{color{blue}{j}}right)tag{2}
    end{align*}




    Comment:




    • In (1) we present the scope of each of the index variables somewhat more clearly by using inner parenthesis and the colors green and blue.


    • In (2) we denote the index variable of the right-most sum with $j$.




    Hint: It is often convenient to give different index variables different names, even if they have no overlapping scope. This usually enhances readability.







    share|cite|improve this answer























      Your Answer





      StackExchange.ifUsing("editor", function () {
      return StackExchange.using("mathjaxEditing", function () {
      StackExchange.MarkdownEditor.creationCallbacks.add(function (editor, postfix) {
      StackExchange.mathjaxEditing.prepareWmdForMathJax(editor, postfix, [["$", "$"], ["\\(","\\)"]]);
      });
      });
      }, "mathjax-editing");

      StackExchange.ready(function() {
      var channelOptions = {
      tags: "".split(" "),
      id: "69"
      };
      initTagRenderer("".split(" "), "".split(" "), channelOptions);

      StackExchange.using("externalEditor", function() {
      // Have to fire editor after snippets, if snippets enabled
      if (StackExchange.settings.snippets.snippetsEnabled) {
      StackExchange.using("snippets", function() {
      createEditor();
      });
      }
      else {
      createEditor();
      }
      });

      function createEditor() {
      StackExchange.prepareEditor({
      heartbeatType: 'answer',
      convertImagesToLinks: true,
      noModals: true,
      showLowRepImageUploadWarning: true,
      reputationToPostImages: 10,
      bindNavPrevention: true,
      postfix: "",
      imageUploader: {
      brandingHtml: "Powered by u003ca class="icon-imgur-white" href="https://imgur.com/"u003eu003c/au003e",
      contentPolicyHtml: "User contributions licensed under u003ca href="https://creativecommons.org/licenses/by-sa/3.0/"u003ecc by-sa 3.0 with attribution requiredu003c/au003e u003ca href="https://stackoverflow.com/legal/content-policy"u003e(content policy)u003c/au003e",
      allowUrls: true
      },
      noCode: true, onDemand: true,
      discardSelector: ".discard-answer"
      ,immediatelyShowMarkdownHelp:true
      });


      }
      });














       

      draft saved


      draft discarded


















      StackExchange.ready(
      function () {
      StackExchange.openid.initPostLogin('.new-post-login', 'https%3a%2f%2fmath.stackexchange.com%2fquestions%2f3004321%2fmultiplying-summation-with-same-indices-and-limits%23new-answer', 'question_page');
      }
      );

      Post as a guest















      Required, but never shown

























      2 Answers
      2






      active

      oldest

      votes








      2 Answers
      2






      active

      oldest

      votes









      active

      oldest

      votes






      active

      oldest

      votes








      up vote
      0
      down vote













      Let's examine series multiplication first:
      $$A=sum_{igeq0}a_i=a_0+a_1+a_2+dots$$
      $$B=sum_{igeq0}b_i=b_0+b_1+b_2+dots$$
      $$AB=big(a_0+a_1+dotsbig)B$$
      $$AB=a_0S_2+big(a_1+a_2+dotsbig)B$$
      $$AB=a_0S_2+a_1S_2+big(a_2+a_3+dotsbig)B$$
      $$AB=a_0S_2+a_1S_2+a_2S_2+big(a_3+a_4+dotsbig)B$$
      This pattern continues:
      $$AB=a_0B+a_1B+a_2B+a_3B+dots$$
      $$AB=sum_{igeq0}a_iB$$
      Now note the following:
      $$a_iB=a_isum_{kgeq0}b_k$$
      $$a_iB=a_ibig(b_0+b_1+b_2+dotsbig)$$
      $$a_iB=a_ib_0+a_ib_1+a_ib_2+dots$$
      $$a_iB=sum_{kgeq0}a_ib_k$$
      Plugging in:
      $$AB=sum_{igeq0}sum_{kgeq0}a_ib_k$$
      Since $i$ is independent of $k$, and they belong to the same set (namely ${xinBbb Z:xgeq0})$, we know that
      $$
      begin{align}
      AB & = a_0b_0+a_0b_1+a_0b_2+dots \
      & + a_1b_0+a_1b_1+a_1b_2+dots \
      & + a_2b_0+a_2b_1+a_2b_2+dots \
      & +dots
      end{align}
      $$

      Which can be greatly abbreviated:
      $$AB=sum_{i,kin S}a_ib_k$$
      Where
      $$S={xinBbb Z:xgeq0}={0,1,2,dots}$$





      Now we can move onto something more related to your problem:
      $$A=sum_{i=0}^{m}a_i$$
      $$B=sum_{i=0}^{m}b_i$$
      These are just like the case above:
      $$AB=sum_{i,kin S}a_ib_k$$
      Where $$S={xinBbb Z:0leq xleq m}$$
      And of course the fact
      $$(1-A)(1-B)=AB-A-B+1$$
      still holds when $A$ and $B$ are series. But one should note:
      $$-A=-a_0-a_1-a_2-dots$$
      and not $$-A=-a_0+a_1+a_2+dots$$






      share|cite|improve this answer

























        up vote
        0
        down vote













        Let's examine series multiplication first:
        $$A=sum_{igeq0}a_i=a_0+a_1+a_2+dots$$
        $$B=sum_{igeq0}b_i=b_0+b_1+b_2+dots$$
        $$AB=big(a_0+a_1+dotsbig)B$$
        $$AB=a_0S_2+big(a_1+a_2+dotsbig)B$$
        $$AB=a_0S_2+a_1S_2+big(a_2+a_3+dotsbig)B$$
        $$AB=a_0S_2+a_1S_2+a_2S_2+big(a_3+a_4+dotsbig)B$$
        This pattern continues:
        $$AB=a_0B+a_1B+a_2B+a_3B+dots$$
        $$AB=sum_{igeq0}a_iB$$
        Now note the following:
        $$a_iB=a_isum_{kgeq0}b_k$$
        $$a_iB=a_ibig(b_0+b_1+b_2+dotsbig)$$
        $$a_iB=a_ib_0+a_ib_1+a_ib_2+dots$$
        $$a_iB=sum_{kgeq0}a_ib_k$$
        Plugging in:
        $$AB=sum_{igeq0}sum_{kgeq0}a_ib_k$$
        Since $i$ is independent of $k$, and they belong to the same set (namely ${xinBbb Z:xgeq0})$, we know that
        $$
        begin{align}
        AB & = a_0b_0+a_0b_1+a_0b_2+dots \
        & + a_1b_0+a_1b_1+a_1b_2+dots \
        & + a_2b_0+a_2b_1+a_2b_2+dots \
        & +dots
        end{align}
        $$

        Which can be greatly abbreviated:
        $$AB=sum_{i,kin S}a_ib_k$$
        Where
        $$S={xinBbb Z:xgeq0}={0,1,2,dots}$$





        Now we can move onto something more related to your problem:
        $$A=sum_{i=0}^{m}a_i$$
        $$B=sum_{i=0}^{m}b_i$$
        These are just like the case above:
        $$AB=sum_{i,kin S}a_ib_k$$
        Where $$S={xinBbb Z:0leq xleq m}$$
        And of course the fact
        $$(1-A)(1-B)=AB-A-B+1$$
        still holds when $A$ and $B$ are series. But one should note:
        $$-A=-a_0-a_1-a_2-dots$$
        and not $$-A=-a_0+a_1+a_2+dots$$






        share|cite|improve this answer























          up vote
          0
          down vote










          up vote
          0
          down vote









          Let's examine series multiplication first:
          $$A=sum_{igeq0}a_i=a_0+a_1+a_2+dots$$
          $$B=sum_{igeq0}b_i=b_0+b_1+b_2+dots$$
          $$AB=big(a_0+a_1+dotsbig)B$$
          $$AB=a_0S_2+big(a_1+a_2+dotsbig)B$$
          $$AB=a_0S_2+a_1S_2+big(a_2+a_3+dotsbig)B$$
          $$AB=a_0S_2+a_1S_2+a_2S_2+big(a_3+a_4+dotsbig)B$$
          This pattern continues:
          $$AB=a_0B+a_1B+a_2B+a_3B+dots$$
          $$AB=sum_{igeq0}a_iB$$
          Now note the following:
          $$a_iB=a_isum_{kgeq0}b_k$$
          $$a_iB=a_ibig(b_0+b_1+b_2+dotsbig)$$
          $$a_iB=a_ib_0+a_ib_1+a_ib_2+dots$$
          $$a_iB=sum_{kgeq0}a_ib_k$$
          Plugging in:
          $$AB=sum_{igeq0}sum_{kgeq0}a_ib_k$$
          Since $i$ is independent of $k$, and they belong to the same set (namely ${xinBbb Z:xgeq0})$, we know that
          $$
          begin{align}
          AB & = a_0b_0+a_0b_1+a_0b_2+dots \
          & + a_1b_0+a_1b_1+a_1b_2+dots \
          & + a_2b_0+a_2b_1+a_2b_2+dots \
          & +dots
          end{align}
          $$

          Which can be greatly abbreviated:
          $$AB=sum_{i,kin S}a_ib_k$$
          Where
          $$S={xinBbb Z:xgeq0}={0,1,2,dots}$$





          Now we can move onto something more related to your problem:
          $$A=sum_{i=0}^{m}a_i$$
          $$B=sum_{i=0}^{m}b_i$$
          These are just like the case above:
          $$AB=sum_{i,kin S}a_ib_k$$
          Where $$S={xinBbb Z:0leq xleq m}$$
          And of course the fact
          $$(1-A)(1-B)=AB-A-B+1$$
          still holds when $A$ and $B$ are series. But one should note:
          $$-A=-a_0-a_1-a_2-dots$$
          and not $$-A=-a_0+a_1+a_2+dots$$






          share|cite|improve this answer












          Let's examine series multiplication first:
          $$A=sum_{igeq0}a_i=a_0+a_1+a_2+dots$$
          $$B=sum_{igeq0}b_i=b_0+b_1+b_2+dots$$
          $$AB=big(a_0+a_1+dotsbig)B$$
          $$AB=a_0S_2+big(a_1+a_2+dotsbig)B$$
          $$AB=a_0S_2+a_1S_2+big(a_2+a_3+dotsbig)B$$
          $$AB=a_0S_2+a_1S_2+a_2S_2+big(a_3+a_4+dotsbig)B$$
          This pattern continues:
          $$AB=a_0B+a_1B+a_2B+a_3B+dots$$
          $$AB=sum_{igeq0}a_iB$$
          Now note the following:
          $$a_iB=a_isum_{kgeq0}b_k$$
          $$a_iB=a_ibig(b_0+b_1+b_2+dotsbig)$$
          $$a_iB=a_ib_0+a_ib_1+a_ib_2+dots$$
          $$a_iB=sum_{kgeq0}a_ib_k$$
          Plugging in:
          $$AB=sum_{igeq0}sum_{kgeq0}a_ib_k$$
          Since $i$ is independent of $k$, and they belong to the same set (namely ${xinBbb Z:xgeq0})$, we know that
          $$
          begin{align}
          AB & = a_0b_0+a_0b_1+a_0b_2+dots \
          & + a_1b_0+a_1b_1+a_1b_2+dots \
          & + a_2b_0+a_2b_1+a_2b_2+dots \
          & +dots
          end{align}
          $$

          Which can be greatly abbreviated:
          $$AB=sum_{i,kin S}a_ib_k$$
          Where
          $$S={xinBbb Z:xgeq0}={0,1,2,dots}$$





          Now we can move onto something more related to your problem:
          $$A=sum_{i=0}^{m}a_i$$
          $$B=sum_{i=0}^{m}b_i$$
          These are just like the case above:
          $$AB=sum_{i,kin S}a_ib_k$$
          Where $$S={xinBbb Z:0leq xleq m}$$
          And of course the fact
          $$(1-A)(1-B)=AB-A-B+1$$
          still holds when $A$ and $B$ are series. But one should note:
          $$-A=-a_0-a_1-a_2-dots$$
          and not $$-A=-a_0+a_1+a_2+dots$$







          share|cite|improve this answer












          share|cite|improve this answer



          share|cite|improve this answer










          answered yesterday









          clathratus

          1,754219




          1,754219






















              up vote
              0
              down vote













              The index variables $k$ are so-called bound variables. This means that their scope (i.e. range of validity) is determined by their sigma-operator $sum$ and the operator precedence rules.




              The following representations are valid
              begin{align*}
              left(1+sum_{k=0}^mx^kright)left(1+sum_{k=0}^my^kright)&=
              left(1+color{green}{left(sum_{k=0}^mx^kright)}right)left(1+color{blue}{left(sum_{k=0}^my^kright)}right)tag{1}\
              &=left(1+sum_{k=0}^mx^kright)left(1+sum_{color{blue}{j=0}}^my^{color{blue}{j}}right)tag{2}
              end{align*}




              Comment:




              • In (1) we present the scope of each of the index variables somewhat more clearly by using inner parenthesis and the colors green and blue.


              • In (2) we denote the index variable of the right-most sum with $j$.




              Hint: It is often convenient to give different index variables different names, even if they have no overlapping scope. This usually enhances readability.







              share|cite|improve this answer



























                up vote
                0
                down vote













                The index variables $k$ are so-called bound variables. This means that their scope (i.e. range of validity) is determined by their sigma-operator $sum$ and the operator precedence rules.




                The following representations are valid
                begin{align*}
                left(1+sum_{k=0}^mx^kright)left(1+sum_{k=0}^my^kright)&=
                left(1+color{green}{left(sum_{k=0}^mx^kright)}right)left(1+color{blue}{left(sum_{k=0}^my^kright)}right)tag{1}\
                &=left(1+sum_{k=0}^mx^kright)left(1+sum_{color{blue}{j=0}}^my^{color{blue}{j}}right)tag{2}
                end{align*}




                Comment:




                • In (1) we present the scope of each of the index variables somewhat more clearly by using inner parenthesis and the colors green and blue.


                • In (2) we denote the index variable of the right-most sum with $j$.




                Hint: It is often convenient to give different index variables different names, even if they have no overlapping scope. This usually enhances readability.







                share|cite|improve this answer

























                  up vote
                  0
                  down vote










                  up vote
                  0
                  down vote









                  The index variables $k$ are so-called bound variables. This means that their scope (i.e. range of validity) is determined by their sigma-operator $sum$ and the operator precedence rules.




                  The following representations are valid
                  begin{align*}
                  left(1+sum_{k=0}^mx^kright)left(1+sum_{k=0}^my^kright)&=
                  left(1+color{green}{left(sum_{k=0}^mx^kright)}right)left(1+color{blue}{left(sum_{k=0}^my^kright)}right)tag{1}\
                  &=left(1+sum_{k=0}^mx^kright)left(1+sum_{color{blue}{j=0}}^my^{color{blue}{j}}right)tag{2}
                  end{align*}




                  Comment:




                  • In (1) we present the scope of each of the index variables somewhat more clearly by using inner parenthesis and the colors green and blue.


                  • In (2) we denote the index variable of the right-most sum with $j$.




                  Hint: It is often convenient to give different index variables different names, even if they have no overlapping scope. This usually enhances readability.







                  share|cite|improve this answer














                  The index variables $k$ are so-called bound variables. This means that their scope (i.e. range of validity) is determined by their sigma-operator $sum$ and the operator precedence rules.




                  The following representations are valid
                  begin{align*}
                  left(1+sum_{k=0}^mx^kright)left(1+sum_{k=0}^my^kright)&=
                  left(1+color{green}{left(sum_{k=0}^mx^kright)}right)left(1+color{blue}{left(sum_{k=0}^my^kright)}right)tag{1}\
                  &=left(1+sum_{k=0}^mx^kright)left(1+sum_{color{blue}{j=0}}^my^{color{blue}{j}}right)tag{2}
                  end{align*}




                  Comment:




                  • In (1) we present the scope of each of the index variables somewhat more clearly by using inner parenthesis and the colors green and blue.


                  • In (2) we denote the index variable of the right-most sum with $j$.




                  Hint: It is often convenient to give different index variables different names, even if they have no overlapping scope. This usually enhances readability.








                  share|cite|improve this answer














                  share|cite|improve this answer



                  share|cite|improve this answer








                  edited 23 hours ago

























                  answered yesterday









                  Markus Scheuer

                  59k454140




                  59k454140






























                       

                      draft saved


                      draft discarded



















































                       


                      draft saved


                      draft discarded














                      StackExchange.ready(
                      function () {
                      StackExchange.openid.initPostLogin('.new-post-login', 'https%3a%2f%2fmath.stackexchange.com%2fquestions%2f3004321%2fmultiplying-summation-with-same-indices-and-limits%23new-answer', 'question_page');
                      }
                      );

                      Post as a guest















                      Required, but never shown





















































                      Required, but never shown














                      Required, but never shown












                      Required, but never shown







                      Required, but never shown

































                      Required, but never shown














                      Required, but never shown












                      Required, but never shown







                      Required, but never shown







                      Popular posts from this blog

                      Can a sorcerer learn a 5th-level spell early by creating spell slots using the Font of Magic feature?

                      Does disintegrating a polymorphed enemy still kill it after the 2018 errata?

                      A Topological Invariant for $pi_3(U(n))$